NCSBN Practice Questions 46-60

Réussis tes devoirs et examens dès maintenant avec Quizwiz!

The 220-pound client was given a bolus dose of heparin and now is to start on a heparin drip. The 500 mL bag of D5W is labeled: "Heparin 25,000 units." The order is 18 units/kg/hr. In mL/hr, how would the nurse set the infusion pump? Round your answer to the nearest whole number.

36 Using dimensional analysis:500 mL/25000 units x 18 units/kg per hour x 1 kg/2.2 pounds = 198000/55000 = 36 mL/1 hrUsing ratio proportion:Convert pounds to kilograms: 220 lb/2.2 = 100 kg18 units/kg = x units/100 kgx = 1800 units each hour500 mL/25000 units = x/1800 units each hourx = 36 mL/1 hr

A nurse in a well-child clinic examines many children on a daily basis. Which of these toddlers requires further follow-up? A. A 30 month-old only drinking from a sippy cup B. A 24 month-old who cries during examination C. A 13 month-old unable to walk D. A 20 month-old only using two- and three-word sentences

A A 30 month-old should be able to drink from a cup without a cover. An approach to use with the months of children is to divide by 12 and think in years.

A client is admitted to the hospital because of heart failure and digoxin toxicity. At home, the client was taking digoxin and furosemide. Which finding should the nurse anticipate on an initial assessment? A. Muscle weakness or cramping B. Blood in the urine C. Confusion D. Tinnitus

A A client taking a nonpotassium-sparing diuretic, such as furosemide, will likely need a potassium supplement to prevent hypokalemia. This client did not take supplemental potassium. Findings of hypokalemia include weakness and muscle cramps. Hypokalemic clients are more sensitive to digoxin toxicity.

A parent of a 14 month-old is sharing concerns with the nurse. Which statement by a parent would alert a nurse to assess for iron-deficiency anemia in the toddler? A. "My child doesn't like many fruits and vegetables, but really loves milk." B. "I can't understand why my child is not eating as much as four months ago." C. "My child doesn't drink a whole glass of juice or water at one time." D. "I know there is a problem since my baby is always constipated."

A About two to three cups of milk a day are sufficient for the young child's needs. Sometimes excess milk intake, a habit carried over from infancy, may exclude many solid foods from the diet. As a result, the child may lack iron and develop a so-called milk anemia. Although the majority of infants with iron deficiency are underweight, many are overweight because of excessive milk ingestion.

The nurse is monitoring a client who is receiving the thrombolytic agent alteplase for treatment of an acute myocardial infarction (AMI). What outcome indicates the client is receiving adequate therapy within the first few hours of treatment? A. Reduction of ST-segment elevation on a 12-lead ECG B. Stabilization of blood pressure C. Absence of cardiac arrhythmias D. Cardiac enzymes are within normal limits

A Alteplase (a t-PA) is used in the management of AMI with ST-segment elevation (STEMI). If thrombolytic therapy was successful, a follow-up ECG will show a reduction of 50% or more in the ST segment. This indicates a return in blood flow to the injured myocardium; however, the ST segment may not return to baseline due to myocardial damage. The other responses are incorrect: successful thrombolysis can cause a variety of cardiac arrhythmias; cardiac enzymes peak 8 hours or more after an AMI; and blood pressure may be unstable.

An adolescent client is hospitalized with hemarthrosis from a hemophilia A bleeding episode. Which order from the provider should be questioned by the nurse? A. Ibuprofen 400 mg as needed for pain B. Immobilization of the joint in a splint C. Desmopressin acetate 0.3 mcg/kg IV infused over 30 minutes D. Passive range of motion every shift

A Aspirin and NSAIDs such as ibuprofen could increase the risk of additional bleeding in this client due to the potential effects on platelet aggregation; therefore, the nurse should contact the provider to clarify the order for pain relief. Hemophilia A results from a deficiency of clotting factor VIII. Desmopressin acetate promotes the release of von Willebrand's antigen from the platelets, which carries factor VIII, helping to control bleeding associated with factor VIII deficiency. Muscle strengthening and range of motion exercises are always indicated. Splinting of joints may provide pain relief.

A client has an order for 1000 mL of D5W to infuse over eight hours. The nurse discovers that 800 mL has infused over the past four hours. What is the priority nursing action at this time? A. Auscultate the lungs bilaterally at the bases B. Check the vital signs and compare to baseline readings C. Place the client in a mid- to high-Fowler's position D. Ask the client if there has been any changes in breathing

A Clients can quickly develop hypervolemia following rapid infusion of isotonic fluids. Findings of fluid overload include increased blood pressure, tachycardia, shortness of breath, crackles heard in the lungs and distended neck veins. The priority nursing action is to auscultate the lungs at the bases. The nurse would also raise the head of the bed and assess vital signs (and compare to previous readings) and heart sounds. The health care provider must be contacted. Using an infusion pump helps prevent situations such as this.

A client is admitted to a psychiatric unit with reports of delusions. What behaviors, if observed by a nurse, would be consistent with delusional thought patterns? A. Suspiciousness and distrust B. Panic and multiple physical complaints C. Flight of ideas and hyperactivity D. Anorexia and hopelessness

A Clinical features of paranoid delusional disorder include extreme suspiciousness, jealousy, distrust, and a belief that others intend to invoke harm. Panic, multiple physical complaints, anorexia and hopelessness may be associated with depression. Flight of ideas and hyperactivity are associated with mania.

A nurse is administering an intravenous piggyback infusion of penicillin. Which client statement would require the nurse's immediate attention? A. "I am itching all over." B. "I have soreness and aching in my muscles." C. "I have cramping in my stomach." D. "I have a burning sensation when I urinate."

A Complaints of itching, feeling hot all over and/or the appearance of raised, red welts on the skin are symptoms of an allergic reaction to the penicillin infusion. Therefore, drug administration should be stopped immediately.

A nurse who is a native English speaker admits an elderly Mexican-American migrant worker after an accident that occurred during work. To facilitate communication the nurse should initially take which of these actions? A. Evaluate the client's ability to speak English B. equest a Spanish interpreter C. Speak to the client through the family or coworkers D. Use pictures, letter boards, or hand gestures

A Despite the cultural heritage, a nurse cannot make assumptions that the client does not understand nor speak English. Stereotyping is to be avoided. The nurse should evaluate if the the client is comfortable with and has an ability to understand or speak English.

The nurse enters the room of a client with a history of asthma who is diagnosed with pneumonia. The client's skin is pink, respirations are 20 per minute, and the client's oxygen is running at 6 liters per minute. What should be the nurse's first action? A. Maintain the current oxygen therapy B. Put the client in more comfortable position C. Lower the oxygen rate to 3 liters per minute D. Call the health care provider about the client's condition

A Even though this client has a history of asthma the condition of pneumonia requires oxygenation. If the oxygen was at too high of a concentration, the hypoxic drive would be eliminated and the client's depth and rate of respirations will decrease. The client's rate is within the higher limits of normal. Thus, there is no need at this time to change the rate of oxygen.

A client, who is nearing death, believes in Hinduism. The nurse should plan for which action? A. At the time of death, the family may request that a priest will trickle milk or holy water into the mouth of the client B. From the moment of death, the body is never left alone until after burial, which usually occurs within 24 hours of death C. The family must be with the client during the process of dying and are responsible to wash the body at least three times after death D. The elders may be with the client during the process of dying and no last rites are given

A Hindu death rituals include trickling a few drops of milk, Ganga or other holy water into the mouth of the person who is dying. The body is prepared for cremation; the body cannot be embalmed or organs removed for donation. The elders of the Church of Jesus Christ of Latter Day Saints (also known as Mormons) are with the client during the dying process and cremation is discouraged. The option of washing the body three times after death (by someone of the same gender, unless the deceased is a minor) is a practice of the Islamic religion. The practices of Judaism include burial within 24 hours after death; some Jewish groups also prohibit autopsy and require a rabbi's pre-approval of organ donation or transplants.

A very active 2 year-old child pulls out a tunneled central venous catheter. What initial nursing action is most appropriate? A. Apply pressure to the vessel insertion site B. Assess heart rate, rhythm and all pulses C. Obtain emergency equipment D. Use cold packs at the exit incision site

A If a central venous catheter is accidentally removed, pressure should be applied to the vein exit site and chest area above it with gauze dressing or a clean washcloth. The primary care provider should be notified. Cold packs are not indicated at this time.

A client, who reports unintended weight loss, drug abuse and night sweats, is admitted with a preliminary diagnosis of HIV/AIDS. Which of these existing clients would be the most appropriate roommate for this newly admitted client? A. Exacerbation of migrating polyarthritis with severe pain B. Acute tuberculosis with a productive cough of discolored sputum for more than three months C. Lupus and vesicles on one side of the middle trunk from the back to the abdomen D. Pseudomembranous colitis and C. difficile

A It is most appropriate to place clients with similar diagnoses in the same room. Because this option does not exist, the nurse would understand that a client with HIV/AIDS would be immunocompromised and should not be placed in a room with any client with an active infection. Of the available options, the client with arthritis would be the best roommate for the client with HIV/AIDS. Typically, standard precautions would be used for a person diagnosed with HIV/AIDS (unless the person presents with cough/fever/pulmonary infiltrate, in which case the person would be placed in a private room and airborne plus contact precautions would be implemented).

The nurse is teaching a 27 year-old client with asthma about the therapeutic regime. Which statement by the client would be incorrect and indicate the need for additional instruction? A. "I need to limit my exercise, especially activities such as walking and running." B. "I should learn stress reduction and relaxation techniques." C. "I should contact the clinic if I am using my medication more often." D. "I should monitor my peak flow every day."

A Limiting physical activity in an otherwise healthy, young client should not be necessary. If exercise intolerance exists, the asthma management plan should include specific medications to treat the problem such as using an inhaled beta-agonist five minutes before exercise. The goal is always to return to a normal lifestyle.

An 8 year-old child is hospitalized with minimal-change disease (MCD). The nurse assists the child to select a lunch menu. Which menu selection is the best choice? A. Grilled chicken strips, corn on the cob with 1 pat of butter, skim milk B. Bologna and cheese sandwich with mustard, vanilla pudding snack, skim milk C. Peanut butter and sliced banana sandwich, apple, skim chocolate milk D. Frankfurter on bun (plain), strawberry gelatin dessert, skim chocolate milk

A MCD is a kidney disease in which large amounts of protein are lost in the urine. Corticosteroids are used to treat the disease; ACE inhibitors and diuretics are used to treat the edema. Treatment also includes eating a healthy, low-sodium diet with high-quality protein. Of the given choices, grilled chicken strips, corn on the cob and a glass of skim milk has the smallest total sodium content (less than 500 mg) and is the healthiest diet. Since nearly every layer of a sandwich is loaded with salt, the bologna and cheese sandwich (with around 1260 mg sodium) and the frankfurter on the bun (717) are not the best choices. However, the peanut butter and sliced banana sandwich, apple and milk option is a close second (about 650 mg sodium.)

A client who is the victim of domestic violence states to the nurse, "If only I could change to be how my partner wants me to be. I know things would be different." Which would be the best response by the nurse? A. "Batterers lose self-control because of their own internal reasons. The violence shown to you is not because of what you did or did not do." B. "No one deserves to be beaten. Are you doing anything to provoke your partner into beating you?" C. "The violence is temporarily caused by unusual circumstances. Don't stop hoping for a change." D. "Perhaps, if you understood the need to abuse, you could stop the violence."

A Only the perpetrator (batterer) has the ability to stop the violence. A change in the behavior of the client will result in a change in the outcomes. However, a change in the client's behavior will not cause the abuser to become less violent or nonviolent. In most cases, this client will need to leave the environment or home. Both the batterer and the recipient will need therapy.

The client is diagnosed with Parkinson's disease (PD) and takes more than one hour to dress for scheduled therapies. Based on this finding, what is the most appropriate nursing intervention? A. Allow the client the time needed to dress B. Encourage the client to dress more quickly C. Ask family members to dress the client D. Demonstrate methods on how to dress more quickly

A Parkinson's disease is a degenerative neurological disorder resulting from nerve cells in the brain not producing enough dopamine, which regulates movement. People with PD experience tremors, muscle stiffness, slow movement, rigidity and poor balance and coordination. With careful planning and activity modification, the client can maintain his ability to safely care for himself. The nurse should plan for and allow enough time for the client to meet his own needs when dressing, toileting and bathing.

A client with paranoid thoughts refuses to eat because of the belief that the food is poisoned. What is the most appropriate statement the nurse can make at this time? A. "Let's see if your partner could bring food from home." B. "If you don't eat, I will have to suggest for you to be tube-fed." C. "The food has been prepared in our kitchen and is not poisoned." D. "Here, I will pour a little of the juice in a medicine cup to drink it to show you that it is OK."

A Reassurance is ineffective when a client is actively delusional. This option avoids both arguing with the client and agreeing with the delusional premise. To suggest that a feeding tube is a possibility offers a logical response to a primarily affective concern. When the client's condition has improved, gentle negation of the delusional premise can be employed.

A nurse is caring for a client who has just been admitted with an overdose of aspirin. The following lab data is available: PaO2 95, PaCO2 30, pH 7.5, K 3.2 mEq/L. Which should be the nurse's first action? A. Assist the client to breathe into a paper bag B. Observe the respiratory rate trends C. Monitor intake and output every hour D. Prepare to administer oxygen by mask

A Side effects of aspirin toxicity include hyperventilation, which can result in respiratory alkalosis in the initial stages. Breathing into a paper bag will prevent further reduction in PaCO2.

Parents call the emergency department to report that their toddler has swallowed drain cleaner. The triage nurse instructs them to call for emergency transport to the hospital and suggests that the parents give the toddler sips of which substance while waiting for an ambulance? A. Water B. Soda C. Lemonade D. Tea

A Small amounts of water will dilute the corrosive substance prior to gastric lavage and is best given within 30 minutes of ingestion of the poison. The other substances have the potential to cause a harmful reaction with the drain cleaner.

While obtaining the history of a 2 week-old infant during the well-baby exam, the nurse finds that the neonatal screening for phenylketonuria (PKU) was done when the infant was less than 24 hours-old. What is the priority nursing action? A. Obtain a repeat blood test at this point B. Document that the test results are pending C. Contact the hospital of birth for the results D. Schedule the infant for a repeat test in two weeks

A Testing for PKU is most reliable when protein has been ingested. A repeat blood specimen must be obtained by the third week of life if the initial specimen was taken from an infant less than 24 hours-old.

An RN who usually works in a spinal rehabilitation unit is reassigned to the emergency department for a shift. Which of these clients should the charge nurse assign to this reassigned RN? A. An adolescent who has been on pain medications for terminal cancer with an initial assessment finding of pinpoint pupils and a respiratory rate of 10 B. An elderly client who reports having taken a "large crack hit" 10 minutes prior to walking into the emergency room C. A middle-aged client who says, "I took too many diet pills" and "my heart feels like it is racing out of my chest." D. A young adult who says, "I hear songs from heaven. I need money for beer. I quit drinking two days ago for my family. Why are my arms and legs jerking?"

A The ED charge nurse should not assign the most critical and acutely ill clients, who require complex and highly-specialized care, to the nurse from spinal rehab. Instead, the ED charge nurse should assign clients whose conditions are more stable. The adolescent (with a chronic condition) is exhibiting findings consistent with narcotic use/abuse/overdose. Even though this is an acute situation, this client's condition is more stable than the others, especially since the effects of the pain medication can be quickly reversed using naloxone. The other clients (the older adult who recently used crack cocaine, someone who is tachycardic or someone who may be experiencing DTs) all have potentially life-threatening conditions.

The client has recently undergone an abdominal hernia repair in a same-day surgery center. The registered nurse (RN) assigns the unlicensed assistive person (UAP) to provide the client with a full liquid diet and provides the UAP with additional instructions about how to minimize client distress when the lunch tray arrives. Which instruction by the nurse is most appropriate to minimize client distress? A. "Encourage the client to eat slowly to prevent gas formation in the intestine." B. "Tell the family they can bring in extra drinks such as Gatorade if the client wishes." C. "Make sure the client gets at least two cartons of milk." D. "Let me know right away if the client is able to eat any of the food on the tray."

A The RN can assign tasks with expected and predictable outcomes. The UAP is given adequate information about the task of what to tell the client and how to promote the best outcome with less distress for the client. Milk is to be avoided because it often results in gas production in some people. To have extra fluids available is not a bad action; however, it does not answer the question of how to minimize distress of the client. It is unlikely that clients in an ambulatory surgical center will have difficulty eating after surgery.

A client calls the clinic and states to the triage nurse: "I had an upset stomach and took Pepto-Bismol and now my tongue looks black. What's happening to me?" What would be the nurse's best response? A. "This is a common and temporary side effect of this medication." B. "How long have you had an upset stomach?" C. "Come to the clinic so you can be seen by the health care provider." D. "Are your stools also black?"

A The best response would be to explain that a dark tint of the tongue is a common and temporary side effect of bismuth subsalicylate (Pepto-Bismol). Although it may also turn stools a darker color, do not confuse this with black, tarry stools, which is a sign of bleeding in the intestinal tract. After addressing the client's initial concern and the reason for the call, the nurse can ask about the upset stomach and then ask the client to come to the clinic if necessary.

A nurse has been teaching a client with type 1 diabetes about insulin. Which statement by the client is incorrect and indicates a need for further teaching? A. "I always make sure to shake the NPH bottle hard to mix it well." B. "I use a sliding scale to adjust regular insulin to my sugar level." C. "Since my eyesight is so bad, I ask the nurse to fill several syringes." D. "I keep my regular insulin bottle in the refrigerator."

A The bottle should by rolled gently, not shaken. The shaking introduces air bubbles that may result in an inaccurate dosage.

The charge nurse is making assignment for the health care team. Which of these tasks can be safely delegated to the licensed practical nurse (LPN)? A. Provide stoma care for a client with a well-functioning ostomy B. Care for a recent complicated double barrel colostomy C. Teach the initial ostomy care to a client and family members D. Assess the function of a newly created ileostomy

A The care of a mature stoma and the application of an ostomy appliance may be delegated to a LPN. The condition of this client is stable, there's a low likelihood of any emergency and care of this client is not too complex. The other options require higher level care by the RN. The RN is the manager of care and is responsible for any initial teaching; the LPN can reinforce information once it has been introduced by the RN.

A 6 year-old child is diagnosed with recurrent urinary tract infections (UTIs). Which one of these instructions would be best for the nurse to tell the caregiver of the child? A. Use plain water for the bath and shampoo the hair last B. Increase bladder tone by delaying voiding C. When laundering clothing, rinse several times D. Have the child use antibacterial soaps while bathing

A The hair should be shampooed last with a rinsing of plain water over the genital area. The oils in soaps and bubble bath can cause irritation, which may lead to UTIs in toddlers, preschoolers and school-aged girls. Antibacterial soap might destroy some normal flora and is not a recommended approach.

The nurse is teaching a client about the use of sublingual nitroglycerin. Which finding should be included as the most common side effect? A. Headache B. Depression C. Dry mouth D. Anorexia

A The most common side effect of nitroglycerin is headache due to generalized vasodilatation. The headaches may gradually become less severe as the client continues to take the medication.

A client exhibiting confusion has been placed in physical restraints by order of the health care provider. Which task could be assigned to an unlicensed assistive person (UAP)? A. Assist the client with activities of daily living B. Document mental status and muscle strength C. Monitor the client's physical safety D. Document client's status every two hours

A The person to whom the activity is delegated must be capable of performing it. The UAP is capable of assisting clients with basic needs and routine tasks. Only the nurse can monitor and evaluate the client's condition.

The nurse is teaching the client scheduled for a thyroidectomy about taking potassium iodide (SSKI, ThyroShield) drops. What information shall the nurse include? A. Mix the medication with juice or milk. B. Take the medication on an empty stomach. C. Store the medication in the refrigerator. D. The medication will enlarge the thyroid gland.

A This medication is used preoperatively, 10-14 days before surgery, to reduce the size and vascularity of the thyroid gland, not enlarge it. Potassium iodide drops should be mixed with water, fruit juice, milk, broth or even formula. The client can use a straw to drink the mixture. To minimize gastrointestinal irritation, it can be given after meals or with food. It should be stored at room temperature.

Upon the return of a client from surgery after an open reduction of a femur fracture, the nurse notes a small bloodstain on the cast and marks it. Four hours later the nurse observes that the stain has doubled in size. What is the best action for the nurse to take? A. Outline the new spot then continue monitoring B. Call the health care provider C. Access the site by cutting a window in the cast D. Record the findings in the client's record

A To make a mark outlining the spot is a good way to monitor the amount of bleeding over a period of time. In addition to outlining the spot, the nurse should note the time and date on the cast. If the bleeding does not appear to be excessive, monitoring the drainage would be appropriate. Some bleeding is expected after this type of surgery. The bleeding should also be documented in the nurse's notes.

The clinic nurse is evaluating an older male client who reports having trouble urinating. After the client uses the bathroom, which method should the nurse use to check for post-void residual (PVR)? A. Scan the bladder, using a portable ultrasound scanner. B. Palpate for rounded swelling above the pubic bone. C. Insert an intermittent urinary catheter (straight catheterization). D. Check for rebound tenderness in the lower abdomen.

A Urinary retention and incomplete bladder emptying can result from urethral obstruction, as seen in benign prostatic hyperplasia (BPH). The nurse can palpate the area from the umbilicus towards the symphysis pubis. An empty bladder rests behind the symphysis pubis and should not be palpable. The nurse can also percuss this area. A urine-filled bladder produces a dull sound. But a bladder ultrasound is the most effective technique since it will digitally register bladder volume. Routine catheterization to check for PVR is not recommended. Abdominal rebound tenderness will not determine urinary retention.

A nurse is caring for a 20-lb (9 kg) 6 month-old infant with a three-day history of diarrhea, occasional vomiting and fever. Peripheral intravenous therapy has been initiated with 5% dextrose in 0.45% normal saline with 20 mEq of potassium per liter infusing at 35 mL/hr. Which finding should the nurse report to the health care provider immediately? A. No measurable voiding in four hours B. Periodic crying and irritability C. Three episodes of vomiting in one hour D. Vigorous sucking on a pacifier

A With no measurable urine output, the infant may experience hyperkalemia, which could occur with continued IV potassium administration because potassium is excreted via the kidneys.

The nurse works with clients in an outpatient substance abuse treatment program. Which intervention is indicated to prevent relapse and promote a successful recovery? (Select all that apply.) A. Medication-assisted treatment B. Participate in group psychotherapy C. Refer clients for mental health assessments D. Discharge clients who fail random drug tests E. Counseling about alternative coping skills

A,B,C,E Treatment for substance use disorder (SUD) includes medications, especially for alcohol and opioid abuse, counseling and group psychotherapy, attendance at mutual help groups, such as Alcoholics Anonymous or Narcotics Anonymous, and learning new ways to cope with cravings and urges to use. Many drug-addicted individuals also have other mental disorders and should be assessed by health care professionals. Like other chronic diseases, relapse is likely and some individuals may fail random drug tests. This doesn't mean that treatment is a failure but that additional modifications and treatment are needed.

Sputum culture results for a client admitted with a cough and fever indicate a methicillin-resistant Staphylococcus aureus (MRSA) infection in the nares. What nursing intervention must now be taken? (Select all that apply.) A. Move the client to a private room B. Place a mask on the client if the client needs to leave the room C. Place the client in a room with another client colonized with MRSA D. Staff will wear N-99 or N-100 particulate respirators when in the client's room E. Dedicate the use of personal and noncritical medical equipment to the client

A,B,C,E When possible, a private room would be best, but cohorting is often used for multidrug resistant organisms such as MRSA. If the client needs to be transported to another area, the client should wear a mask, especially if there's a productive cough. Staff should practice excellent hand hygiene and other standard precautions, but a respirator is not needed for MRSA in the nares. To minimize the risk of spreading infection, equipment or personal items should kept in the client's room and dedicated for his/her use.

A client has been taking rosuvastatin for six weeks as part of a treatment plan to reduce hyperlipidemia. The clinic nurse is reviewing and reinforcing information about the medication with the client. Which statement by the client indicates an understanding about the medication? (Select all that apply.) A. "I will need to call my doctor if I have any muscle weakness or pain, especially in my legs." B. "I will need to come back to have my liver and kidney labs checked." C. "I need to be careful when I get up because this medication can make my blood pressure drop." D. "I add some nuts and fresh fruit to my oatmeal in the morning and I can't remember when I last ate a steak." E. "This medication has to be taken first thing in the morning, before I eat breakfast."

A,B,D Clients taking rosuvastatin need to be monitored for alteration in liver function. An adverse effect of rosuvastatin is muscle pain and weakness (rhabdomyolysis). Left untreated, rhabdomyolysis can lead to renal impairment. The medication does not affect blood pressure or cause orthostatic hypotension. The client should be taught to follow a low-cholesterol diet, which includes increasing intake of whole grains and limiting intake of foods high in saturated fats, trans fats and dietary cholesterol. The medication is ordered once a day. The client can take it at any time of day, preferably at the same time of day each day, before or after eating.

The nurse needs to start an IV on an oriented adult. Which supplies will the nurse select? (Select all that apply) A. IV starter kit B. IV catheter C. Rolled gauze D. Saline flush or ordered solution E. Labels for insertion site and tubing F. Arm board

A,B,D,E An arm board or rolled gauze are only necessary for a person that cannot keep the arm straight or picks at the IV site, often because of confusion or other altered level of consciousness. All the other supplies listed are standard.

A client is diagnosed with rheumatoid arthritis (RA). Which types of drugs might the nurse expect to be ordered as a combination drug therapy regimen? (Select all that apply.) A. Glucocorticoids B. Biological-response modifiers C. Antimicrobial agents D. Diuretics E. Anti-inflammatory drugs

A,B,E Rheumatoid arthritis is a chronic, systemic autoimmune disorder that results in symmetric joint destruction. Research shows that multiple drug therapy is most effective in protecting against further destruction and promoting function. Analgesics and anti-inflammatory drugs are used. Disease-modifying anti-rheumatic drugs (DMARDs) such as methotrexate help slow or stop progression of RA. Biological response modifiers are used to help stop inflammation. Glucocorticoids can also be used for severe RA or when RA symptoms flare to ease the pain and stiffness of affected joints. Because RA is not an infectious disease, antimicrobials are ineffective. Although there is swelling in the joints, it is not fluid, so diuretics are not part of the treatment plan.

The nurse is caring for a client with chronic renal failure who is undergoing peritoneal dialysis. The nurse notes that the dialysate solution is instilling very slowly. Which of the following actions would be appropriate for the nurse to implement? (Select all that apply.) A. Reposition the client B. Assess for headache and hypertension C. Assess for bruit or vibration D. Check tubing and catheter for kinks

A,D Slow dialysate instillation may be due to a partially obstructed tube or catheter. Checking for kinks and repositioning may facilitate improved instillation of this fluid. Assessment for bruit or headache and hypertension are appropriate for hemodialysis situations, not peritoneal dialysis.

A client is newly diagnosed with hypothyroidism and is prescribed levothyroxine 50 mcg/day by mouth. As part of the teaching plan, what should the nurse emphasize about this medication? A. It can be taken with an antacid if stomach upset occurs B. It should be taken in the morning C. It must be stored in a dark container D. It may decrease the client's energy level

B A thyroid supplement should be taken in the morning on an empty stomach with 8 ounces of water to maximize effects. Also, the client should avoid foods high in fiber, iron or soybeans within four hours of taking this medication because they may interfere with this drug's absorption. The medication should not be given in the evening or prior to bedtime because it may cause insomnia.

A client is diagnosed with amyotrophic lateral sclerosis (ALS). Which nursing action will help prevent the complications of atelectasis and pneumonia in this client? A. Use of the incentive spirometer every two hours while awake B. Chest physiotherapy twice a day C. Active and passive range of motion exercises twice a day D. Repositioning every two hours around the clock

B ALS is a progressive neurodegenerative disease. Early symptoms include increasing muscle weakness, especially involving the arms and legs, speech, swallowing or breathing; eventually there is total paralysis. As the chest muscles and diaphragm become weaker, it will be more difficult to cough and clear secretions. This will predispose clients to develop recurrent pulmonary infections. Chest physiotherapy and airway clearance devices will help prevent and/or treat these secondary infections. Deep breathing and coughing exercises may not be realistic for this client. Repositioning is targeted to circulation issues and is not specific for prevention of complications associated with the lung.

The nurse is providing teaching to a client who has been prescribed cyclophosphamide for breast cancer treatment. Which of the following statements made by the client would indicate that additional teaching is needed? A. "I will probably need to plan on using a wig to cover my hair loss." B. "I should limit the amount of fluids I drink while taking this medication." C. "I will need to stay away from children when my white blood cell count is low." D. "I may have trouble getting pregnant due to the damaging effects of the medication."

B All statements are true except that increased fluid intake is recommended, not limiting fluid intake. Increased fluid intake will help eliminate the chemotherapeutic agent from the body.

A child is admitted to the unit with the suspected diagnosis of pertussis (whooping cough). What is the priority nursing intervention for this child? A. Maintain hydration and encourage fluids B. Implement droplet precautions C. Monitor respiratory rate and oxygen saturation D. Initiate anti-infective therapy

B Although all the responses are appropriate nursing interventions, the priority is to implement strict droplet precautions, in addition to standard precautions. Pertussis is highly contagious and is spread through close contact.Therapeutic management focuses on providing respiratory support and eradicating the bacterial infection (macrolides, such as erythromycin, are the drug of choice). Administer fluids and keep the client hydrated to help thin secretions. It is also important to monitor the client's heart rate, respiratory status and oxygen saturation, especially during coughing paroxysms.

The client is trembling and appears fearful as he enters a locked behavioral health unit for the first time. Which initial action by the nurse would be best for the client? A. Ask a mental health technician to take the client to his room and measure vital signs B. Introduce self to the client and accompany the client to his room C. Give the client orientation information and review the unit rules and regulations D. Take the client to the community day room and introduce him to everyone

B Anxiety is triggered by change that threatens an individual's sense of security. The nurse should remain calm, minimize stimuli and move the client to a safer and quieter setting. This client is not ready to interact with others or listen to unit rules. It would not be the correct response to assign care to another staff person.

A nurse observes a staff member caring for a client who is postop for a left unilateral mastectomy. The nurse should intervene if the staff member is observed doing which action? A. Elevating the client's left arm above heart level B. Taking the blood pressure in the left arm C. Reinforcing the client to restrict sodium intake D. Compressing the drainage device

B Clients who have had a unilateral mastectomy should not have their blood pressure measured on the affected extremity. This helps avoid the possibility of lymphedema and tissue trauma postoperatively and in the future.

A nurse is performing an assessment on a client with pneumococcal pneumonia. Which finding should the nurse anticipate? A. Hacking, nonproductive cough B. Bronchial breath sounds in the outer lung fields C. Decreased tactile fremitus D. Hyper-resonance of areas of consolidation

B Consolidated lung tissue contains exudate from the infectious process, changing the infected portion of the lung tissue from air-filled to fluid-filled, and increasing the ability of the affected lung field to transmit sounds. Consolidated lung tissue transmits bronchial breath sounds to outer lung fields.

A nurse is assigned to a 12 year-old diagnosed with an acute illness. Which approach indicates that the nurse understands common sibling reactions to hospitalization? A. The siblings may enjoy privacy B. Visitation is helpful for both C. Younger siblings adapt very well D. Those cared for at home cope better

B Contact with the ill child helps siblings understand the reasons for hospitalization and maintains their relationships.

The client is admitted with the diagnosis of diabetic ketoacidosis (DKA). Which serum lab finding should alert the nurse that immediate action is required? A. Potassium 5.0 mEq/L (5 mmol/L) B. Hematocrit 60% (0.6) C. PaO2 = 79 mm Hg D. pH = 7.34

B DKA is defined as a blood pH less than 7.30, bicarbonate level less than 18 mEq/L (18 mmol/L), and blood glucose greater than 250 mg/dL (13.9 mmol/L). The potassium for this client is normal high and PaO2 level for arterial blood is near the low end of normal (80 -100 mm Hg). Hyperglycemia induces osmotic diuresis, causing water and electrolyte loss (especially potassium) and the high hematocrit confirms severe dehydration. The single most important therapy for DKA is IV fluid administration, usually with added potassium, to lower blood glucose levels and normalize the pH.

The nurse is teaching about precautions for suicide prevention to the parents of an adolescent who recently attempted suicide. During the discussion, the nurse should describe which of these behavioral cues as an indication for intervention? A. Experiencing the breakup with a boyfriend B. Giving away valued personal items C. Exhibiting angry outbursts at significant others D. Verbalizing fear of being left alone

B Eighty percent of all potential suicide victims give some type of indication that self-destructiveness is being considered and should be addressed. Two types of cues include physical, which is the giving away of prized possessions, and mood, which is a sudden change to happiness or joy. Either cues can lead one to suspect that a client is having suicidal thoughts or is developing a plan. Thus, they should alert family or professionals to ask the question: Do you have a plan for suicide? Or what is your plan for self-destruction?

A nurse is evaluating the quality of home care for a client diagnosed with Alzheimer's disease. It would be a priority to reinforce which statement made by a family member? A. "To take the medication three times a day is not a problem." B. "We have safety bars installed in the bathroom and have alarms on the exterior doors." C. "At least two full meals a day should be eaten." D. "We go to a group discussion every week at our community center."

B Ensuring safety of the client with increasing memory loss is a priority of home care. Note that all of the options are positive statements for the care of this client. However safety is the most important item to reinforce.

A client is being discharged home today and will be taking potassium (K-Dur) 20 mEq per day by mouth. What should the nurse advise the client to avoid due to its effects of lowering serum potassium levels? A. Occasional nonsteroidal anti-inflammatory drug (NSAID) use B. Frequent daily snacks of black licorice C. Prescribed potassium-sparing diuretic D. Food seasoned with salt substitute

B Excessive intake of black licorice can lead to decreased serum potassium due to the effect of glyceric acid (aldosterone effect). The excessive use of salt substitutes (which usually contain potassium chloride), potassium-sparing diuretics and NSAIDs have the potential for raising potassium levels.

A nurse is teaching parents of a 7 month-old about adding table foods. Which option is an appropriate finger food? A. Popcorn with minimal kernels B. Sliced bananas cut vertically C. Hot dog pieces cut in short pieces D. Whole purple and white grapes

B Finger foods should be bite-size pieces of soft food such as bananas at this age. Hot dogs if cut horizontally and grapes can accidentally be swallowed whole and can occlude the airway. Popcorn is too difficult to chew at this age and can irritate the airway if swallowed.

A nurse is performing chest compressions on an adult client in cardiac arrest. Another nurse enters the room in response to the call for help. What should happen next? A. The second nurse will take 10 seconds to check the pulse and "look, listen and feel" for breathing B. The second nurse will maintain an open airway and perform ventilations while the first nurse continues chest compressions C. The first nurse will now perform 15 chest compressions and then wait while the second nurse performs 2 ventilations D. The second nurse will leave to get personal protective devices (PPD) and oxygen

B In two-rescuer CPR, the first rescuer performs chest compressions and the second rescuer performs (bag-mask) ventilation. Rescuers should switch roles after five cycles (about every two minutes). The compressing rescuer should give (30) continuous chest compressions at a rate of at least 100 per minute without pauses for ventilation; the rescuer will provide a breath every six to eight seconds. Rescuers should all have quick access to personal protective devices (PPD) prior to any intervention.

A nurse is assessing a 9 year-old child after several days of treatment for a documented strep throat. Which statement is incorrect and suggests that further teaching is needed? A. "Sometimes I take my medicine with fruit juice." B. "Sometimes I take the pills in the morning and other times at night." C. "I am feeling much better than I did last week." D. "My mother makes me take my medicine right after school."

B Inconsistency in taking the prescribed medication indicates more teaching is needed. Medications for infections need to be given at regular intervals around the clock.

A client has severe mental illness. The nurse should know that a priority action for involuntary hospitalization of the client diagnosed with severe mental illness involves which of the following goals? A. Return to independent functioning B. Protection from harm to self or others C. Elimination of findings D. Reorientation to reality

B Involuntary hospitalization may be required for persons considered dangerous to self or others or for individuals who are considered gravely disabled. The other options are goals associated with other situations of less severe diagnoses of mental illness.

The nurse is caring for a 75 year-old client with peripheral arterial insufficiency of the lower extremities. Which intervention should be included in the plan of care to reduce leg pain? A. Support smoking cessation efforts B. Lower the legs to a dependent position C. Apply cold compresses D. Elevate the legs above the heart

B Ischemic pain is relieved by placing feet in a dependent position. This position improves peripheral perfusion. Smoking cessation will prevent long-term progression of the disease, but will not reduce the pain the client is presently experiencing.

The client, who is diagnosed with a musculoskeletal disorder, ambulates with an orthotic. The nurse assigns several tasks or activities to the unlicensed assistive personnel (UAP) for this client. Which activity cannot be assigned to the UAP? A. Assist the client transfer from a bed to a chair B. Monitor the client's response to ambulatory activity C. Encourage the client's independence in self-care D. Report redness or signs of skin breakdown around the orthotic

B Nurses cannot delegate any part of the nursing process. Monitoring the client's response to interventions requires evaluation, a task that can only be performed by the RN. The other options can be assigned and performed independently by the UAP.

A 35 year-old client with sickle cell crisis is talking on the telephone but stops as the nurse enters the room to request something for pain. The nurse should take which of these actions? A. Recommend relaxation exercises for pain control B. Administer the prescribed analgesic C. Encourage an increased fluid intake D. Administer a placebo if possible

B Pain, especially chronic pain, may be present even without overt signs. Thus, the nurse is to accept that the client is in pain if the client identifies such. Relief of pain is a priority expected outcome for treatment of sickle cell crisis.

The parents of a child who has suddenly been hospitalized for an acute illness state that they should have taken the child to the pediatrician earlier. Which approach should the nurse use when dealing with the parents' comments? A. Explain the cause of the child's illness B. Accept their feelings without judgment C. Acknowledge that early care would have been better D. Focus on the child's needs and recovery

B Parents often blame themselves for their child's illness. Feeling helpless and angry is normal and these feelings must be accepted. The other actions are inappropriate at this time.

The 55 year-old female is scheduled for abdominal surgery. Which factor in the client's history indicates that the client is at risk for thrombus formation in the postoperative period? A. 10 percent less than ideal body weight for past year B. Estrogen replacement therapy for the past three years C. Hypersensitivity to heparin 20 years ago D. History of acute hepatitis A

B Post-menopausal women using hormone replacement therapy have a higher risk of deep vein thrombosis and pulmonary embolism. The estrogen in hormone replacement therapy (and in birth control pills) can increase clotting factors in the blood, especially if the woman is a smoker and/or overweight. The other information in the client's history is unremarkable for postoperative complications.

A nurse is caring for a client diagnosed with an exacerbation of rheumatoid arthritis. Which nursing diagnosis should the nurse give as a priority in the plan of care? A. Risk for injury B. Alteration in comfort C. Risk for an alteration in mobility D. Self-care deficit

B Relief of the pain is the priority objective for a client in an exacerbation of rheumatoid arthritis. The second nursing diagnosis of importance is self-care deficit. Then the risk for an alteration in mobility and injury would follow with both being associated with each other.

After the death of a client, the family approaches the nurse and requests that a family member be allowed to perform a ritual bath on the deceased client prior to moving the body. What would be the most appropriate response by the nurse? A. "A ritual bath will have to wait until after postmortem care." B. "Is there anything you need from me to perform the bath?" C. "I will have to check on hospital regulations and policies." D. "These procedures have to be carried out by our staff."

B Rituals are processes that allow the bereaved to acknowledge the reality of death. Religious rituals, specifically, offer meaning and provide hope within the context of the particular faith tradition. The nurse should inquire about the family's wishes for rituals or observances following death and respect the family's request. The other options are inappropriate and culturally insensitive.

The nurse is assessing a 72 year-old client with a full-leg cast on his left leg three days after cast application and finds bilateral pedal edema. Based on this finding, what condition should the nurse consider? A. Compartment syndrome B. Heart failure C. Infection D. Thrombophlebitis

B Swelling after injury or surgery and reduction usually peaks within 24 to 48 hours, with only minimal swelling expected afterwards. If the client had pedal edema only on the casted leg, the nurse should consider extension of the initial injury/trauma, compartment syndrome, or thrombophlebitis. However, with bilateral pedal edema, the nurse should consider right-sided heart failure.

A nurse is teaching a group of college students about breast self-examination when a student asks for the best time to perform the monthly self-exam. What is the best reply by the nurse? A. "Do the exam at the same day and time every month." B. "Right after the period ends, when your breasts are less tender." C. "The first of every month, because it is easiest to remember." D. "Ovulation, or mid-cycle is the best time to detect changes."

B The best time for a breast self-exam (BSE) is one week or seven days after the first day of a menstrual cycle. This is when the breasts are no longer swollen or tender from hormonal elevation.

A client was admitted to the psychiatric unit after complaining to friends and family that the neighbors bugged the client's home in order to hear all of the client's business. The client remains aloof from other clients in the unit, paces the floor, and believes that the hospital is a "house of torture." What approach should the nursing interventions take? A. Convince the client that the hospital staff is trying to help B. Provide interactions to assist the client to build trust in the staff C. Help the client to enter into group recreational activities D. Arrange the environment to limit the client's contact with other clients

B The establishment of trust facilitates the clients who exhibit paranoia to feel safer and facilitates a therapeutic alliance between staff and client. Group activities are inappropriate for people who exhibit paranoid behaviors or thoughts. To convince the client that the staff is being helpful is too general of a statement to be the correct answer.

A nurse is recalling the steps in the nursing process. During the evaluation phase for a client, the nurse should focus on which aspect? A. Select interventions that are measurable and achievable within selected timeframes B. The client's status, progress toward goal achievement, and ongoing reevaluation C. Setting short- and long-term goals to ensure continuity of care from hospital to home D. Findings of physical and psychosocial stressors of the client and in the family

B The evaluation step of the nursing process focuses on the client's status, progress toward goal achievement and ongoing reevaluation of the plan of care. The other possible answers focus on some of the other steps of the nursing process.

The nurse is caring for a client diagnosed with a venous stasis ulcer on one leg. Which nursing intervention would be most effective to promote healing? A. Begin proteolytic debridement within 24 hours B. Improve the client's nutritional status C. Apply dressings with the use of sterile technique D. Initiate whirlpool bath therapy

B The goal of clinical management in a client diagnosed with venous stasis ulcers is to promote healing. This can only be accomplished with proper nutrition. The other interventions are appropriate, but without proper nutrition, they would be of little help. Venous ulcers take a long time to heal so proper nutritional therapy is the most important intervention.

The nurse in the preoperative clinic is caring for a client with breast cancer and providing preoperative teaching a week before surgery. The client expresses an interest in complementary and integrative health therapies. Which of the following statements is the most appropriate response by the nurse? A. "Complementary and integrative health therapies are not recommended and you should focus on the upcoming surgery." B. "I will set up a time before the surgery is scheduled for you to talk to your health care providers about complementary and integrative health therapies." C. "You may use any complementary or alternative therapies that you wish to after you recover from surgery." D. "You may want to do some Internet research on the different nutritional therapies that are recommended for your type of cancer."

B The nurse needs to be nonjudgmental and provide an opportunity for the client to ask questions about complementary and integrative health therapies; the nurse should not recommend any specific treatment.

The home health nurse is visiting a client recently discharged after an episode of acute pyelonephritis. Which nursing action should take the highest priority? A. Review oral intake with the client B. Observe client findings for the effectiveness of antibiotic therapy C. Check blood pressure D. Follow up on lab values before the visit

B The priority nursing action is to determine whether the antibiotic therapy has been effective in treating this serious kidney infection. Fever, flank pain, nausea and vomiting would be indicators that the antibiotic therapy has not been effective, requiring contact with the provider for further treatment orders.

The client is prescribed tiotropium to help manage the symptoms of chronic obstructive pulmonary disease. What information should the nurse include when teaching the client about tiotropium? A. It may be a few days before you feel the full effects of tiotropium B. This medication cannot be used to relieve sudden breathing problems C. Be sure to swallow the capsules with a full glass of water D. A common side effect is nausea and loose stools

B Tiotropium is a long-acting anticholinergic bronchodilator. The medication comes as a capsule to use with a specially designed inhaler - clients should never swallow the capsules. For new prescriptions, it's important to tell the client that s/he may start breathing better with the full dose but it may take a few weeks to feel the full effects. It cannot be used as a fast-acting inhaler. Due to its anticholinergic properties, it may cause constipation (not loose stools).

A nurse is assigned to triage clients who are brought to the emergency department after a bus accident. Which client would be cared for first? A. The 9 year-old with dilated pupils and cessation of breathing 10 minutes ago B. The 18 year-old who is diaphoretic and tachycardic and has a board-like abdomen C. The 35 year-old with an abrasion on the right anterior and lateral side of the chest D. The 56 year-old with external rotation and shortening of the left lower extremity

B Triage categories used during a mass casualty incident often use color-coding to prioritize care. The person in greatest need of immediate care (category - red) is the person with a board-like abdomen because this would indicate internal bleeding. The next person requiring urgent care (category - yellow) would be the individual with the shortened leg that is externally rotated, indicating a probable broken hip. The person with the chest abrasions is not currently having any difficulty breathing and can have treatment delayed for hours or even days (category - green). The individual who stopped breathing 10 minutes ago and whose pupils are fixed and dilated would not be treated because this person has already died (category - black).

The registered nurse (RN) is making staffing assignments at the start of a new shift. The RN can assign the care of which of these clients to the licensed practical nurse (LPN)? A. A preoperative client with a history of asthma awaiting an adrenalectomy B. A client with a diagnosis of peripheral vascular disease (PVD) with an ulceration of the lower leg C. A new admission with a history of diagnosis of transient ischemic attacks and syncope D. An older adult client with a diagnosis of hypertension and self-reported noncompliance

B When compared to the other clients, the client diagnosed with PVD is the most stable. This client also has a chronic condition that needs supportive care, as opposed to newly diagnosed conditions that require more in-depth assessment, care and/or teaching. The LPN has a limited role in each of the steps of the nursing process (assessment, planning, implementation and evaluation) and cannot perform any of them independently.

The nurse is caring for a client with an intravenous (IV) infusion for pain control. The nurse should ask an unlicensed assistive personnel (UAP) to do which action? A. Check the IV site for drainage and loosen tape when in the room B. Assist the client with ambulation after supervising a gown change C. Readjust the rate on the pump by 2 mL/minute D. Monitor the client for the degree of pain relief

B When giving assignments to a UAP, the nurse should communicate clearly and specifically what the task is, what should be reported to the nurse, and when it should be reported. Implementation of routine tasks with expected outcomes should be delegated to UAPs. The other options are actions that PNs or RNs could do.

A nurse is working with a new admission who has a baseline blood pressure (BP) of 120/80 mm Hg with a pulse of 66 BPM. This client experiences a drop in BP to 100/40. Which of the following pulse rates would serve to maintain the client's baseline cardiac output (CO)? (Select all that apply.) A. 62 BPM B. 68 BPM C. 70 BPM D. 64 BPM

B,C CO = HR x SV (cardiac output is a function of heart rate times the stroke volume (measured as blood pressure)). Thus, if blood pressure drops, the heart rate must increase to maintain cardiac output.

A 32 year-old female with human epidermal growth factor receptor 2-positive (HER2-positive) metastatic breast cancer is scheduled to begin therapy with pertuzumab. What information is important for the nurse to reinforce and discuss with the client? (Select all that apply.) A. Other therapies for cancer treatment are no longer needed B. Use contraception during and for 6 months following the use of this drug C. Take the medication at the same time every day on an empty stomach D. Report shortness of breath, lightheadedness, dizziness, cough, or swelling of the feet E. Report chills, fatigue, or headache during treatment

B,D,E Pertuzumab (Perjeta) is used in combination with trastuzumab (Herceptin) as a targeted therapy for HER2+ metastatic breast cancer; these meds are used in combination with chemotherapy and radiation. The most common side effects are fatigue, loss of taste, muscle pain, and vomiting; sometimes slowing the infusion rate can help. It is best to eat a small meal before receiving the infusion. Serious side effects include birth defects and fetal death; women of child-bearing age must use a form of effective contraception during and for 6 months following treatment. Drugs that block HER2+ activity decrease left ventricular ejection fraction (LVEF) and will worsen symptoms of congestive heart failure; heart function must be tested before and monitored during treatment.

The nurse is providing discharge teaching for a client with a long leg cast. During instructions, the nurse should recommend which of these exercises for the affected extremity? A. Aerobic B. Range of motion C. Isometric D. Isotonic

C A nurse should instruct the client on isometric exercises for the muscles of the casted extremity. This means the client should be instructed to alternately contract and relax muscles without moving the affected part. The client should also be instructed to do active range of motion exercises for every joint that is not immobilized at regular and frequent intervals of at least every four hours.

A nurse enters a client's room to discover that the client has no pulse or respirations. After calling for help, what is the first action the nurse should take? A. Obtain the crash cart B. Start a peripheral IV C. Initiate chest compressions D. Establish an airway

C According to the 2010 American Heart Association BLS for Health Care Provider guidelines, the nurse should begin with chest compressions after determining unresponsiveness and calling for help. After 30 chest compressions, the nurse would then open the airway and give 2 breaths (C-A-B).

The nurse needs to make frequent detailed client assessments for pressure sore risk. Which client has the highest risk for developing a pressure sore? A. A 40 year-old wearing a controlled ankle motion walker following surgical repair of a ruptured tendon B. A 75 year-old diagnosed with peripheral vascular disease and needs assistance to walk C. A 68 year-old with left-sided paresthesia who is incontinent of urine D. A 55 year-old in balanced-skeletal traction for a fractured femur

C According to the Braden Scale, the risk for developing pressure sores is rated on a scale of 1 to 4 for each of these factors: sensory perception, moisture, activity, mobility, nutrition, and friction and shear; a score of 12 or lower represents a high risk. With all other risk factors being equal, the 68 year-old client with paresthesia and who is incontinent of urine has the greatest risk for developing a pressure sore.

An infant has just returned from surgery for placement of a gastrostomy tube as an initial treatment for a diagnosis of tracheoesophageal fistula (TEF). The mother asks, "When can the tube be used for feeding?" A nurse should respond with which of these comments? A. "The feeding tube can be used immediately." B. "Feedings can begin in five to seven days." C. "The stomach contents and air must be drained first." D. "Healing of the incision must be complete before feeding."

C After surgery for gastrostomy tube placement, the catheter is left open and attached to gravity drainage for 24 hours or more.

The charge nurse is making assignments for a registered nurse (RN), a licensed practical nurse (LPN) and an unlicensed assistive person (UAP). Which assignment would make best use of the LPNs skills and abilities? A. Test a stool specimen for occult blood B. Assist with ambulating a client for the first time after surgery C. Irrigate a wound and reapply a dressing D. Admit a client from the emergency department

C Although LPNs learn about the nursing process and can assess clients, the role of the LPN is typically supportive. The RN would be responsible for admitting a client and for assessing the client's ability to stand and walk after surgery. Both UAPs and LPNs can collect specimens; however the UAP cannot perform procedures requiring sterile technique. Therefore, the UAP could test the stool for occult blood and the LPN would perform the dressing change.

The nurse is reviewing lab values for a client. Which abnormal serum lab value should the nurse anticipate to stay the same during hemodialysis? A. Hypernatremia B. Hyperkalemia C. Low hemoglobin D. High creatinine

C Although hemodialysis improves or corrects electrolyte imbalances such as sodium, potassium and magnesium it has no effect on changing the red cells in conditions such as anemia. Waste products such as creatinine are expected to be lowered with hemodialysis.

A 14 year-old with a history of sickle cell disease is admitted to the hospital with a diagnosis of vaso-occlusive crisis. Which statement by the client would most likely indicate the cause of this crisis? A. "I have really been working hard practicing with the debate team at school." B. "I knew this would happen. I've been eating too much red meat lately." C. "I used cold medicine last week and I have gotten worse." D. "I really enjoyed my fishing trip yesterday. I caught two fish."

C Any condition that increases the body's need for oxygen or alters the transport of oxygen, including infection, dehydration or even cold weather (due to vasospasm) may result in a sickle cell crisis. Sickle cell crisis is also called vaso-occlusive crisis and pain is the most distinguishing clinical feature. Clients should recognize the earliest signs of a vaso-occlusive crisis and seek help, treat all febrile illness promptly, and identify environmental hazards that may precipitate a crisis.

Two staff members call in sick on the medical unit and no additional help is available. The team consists of a registered nurse (RN), a licensed practical nurse (LPN) and an unlicensed assistive person (UAP). Which of these activities can the RN assign to the UAP? A. Perform dressing changes B. Measure vital signs C. Provide basic hygiene care D. Help prepare clients for discharge

C Basic and routine client care, such as providing basic hygiene care, can be assigned to the UAP. The RN and LPN can measure vital signs as part of their initial client assessments - this data is needed in order to make decisions about client care. UAP do not perform dressing changes. Only the RN can prepare clients for discharge.

A nurse is preparing to administer morning medications to a 12 year-old client with heart failure. The child's morning lab values are: sodium 142 mEq/L (142 mmol/L), potassium 2.9 mEq/L (2.9 mmol/L), digoxin level 1.4 ng/mL. Which of the following medications should the nurse not administer until after speaking with the health care provider? A. Spironolactone B. Carvedilol (Coreg) C. Digoxin (Lanoxin) D. Ferrous sulfate

C Because the potassium levels are low (normal is 3.5 to 5 mEq/L or 3.5 to 5 mmol/L), the nurse should not give the digoxin; hypokalemia can predispose a person to digoxin toxicity. The other medications can be administered. Although carvedilol can increase plasma digoxin concentration, the digoxin level is normal. Spironolactone is a potassium-sparing diuretic and because the potassium level is low, this too can be given. Ferrous sulfate does not affect the given lab values.

A nurse is caring for a one-day postoperative client with a T-tube following a cholecystectomy. The nurse would expect which color of drainage from the client's T-tube at this time? A. Light lime greenish B. Dark chocolate brown C. Yellowish brown D. Yellowish red

C Bile, which is brown or yellowish, is the expected drainage from a T-tube. Bile is similar in color to bilirubin. The gastric contents are a greenish color.

A postpartum client admits to frequent alcohol use throughout the pregnancy. Which newborn assessment finding does the nurse associate with fetal alcohol syndrome (FAS)? A. Growth retardation B. Organ dysfunction C. Craniofacial abnormalities D. High birth weight

C Characteristic facial abnormalities are seen in the newborn with FAS, including small head circumference, smaller eye openings, flattened cheekbones and indistinct philtrum. Newborns often have a low birth weight (not high birth weight). Other irreversible effects of alcohol exposure during pregnancy include mental retardation and delayed development; heart defects and vision difficulties or hearing problems; learning disorders; and behavior problems.

A nurse administers cimetidine to a 79 year-old male with a gastric ulcer. Which parameter may be affected by this drug and should be closely monitored by the nurse? A. Blood pressure B. Liver enzymes C. Mental status D. Hemoglobin

C Clients who are over the age of 50 or are severely ill may become temporarily confused while taking H2-receptor blockers, especially cimetidine (Tagamet).

A client is admitted with the diagnosis of pulmonary embolism (PE). While taking a history, the client says: "I was admitted for the same thing twice in the past six months. In fact, the last time was just three months ago." The nurse should anticipate a need for education may be centered around which approach to treatment? A. Thrombolytic therapy B. Increase in the anticoagulant dosage C. Vena cava filter D. Pulmonary embolectomy

C Clients with recurrent PE or those with excessive clotting complications related to medical therapy may require vena cava interruption. This is the placement of a filter device in the inferior vena cava. A filter can be placed transvenously to trap clots before they travel into the pulmonary circulation.

When teaching new parents prevention of sudden infant death syndrome (SIDS) what is the most important practice a nurse should instruct them to do? A. Obtain a home monitor system to hear the sounds from the infant B. Do not allow anyone to smoke in the home C. Place the infant in a supine position for sleep D. Follow recommended immunization schedule

C Current thinking is that infants become hypoxic when they sleep because of positional narrowing of the airway and respiratory inflammation. Sleeping in the prone position may cause oropharyngeal obstruction or affect the thermal balance or arousal state. Sleep apnea is not the cause of SIDS. Because of research findings and the "Back to Sleep" campaign, the incidence of sleep apnea and the number of SIDS deaths have dropped dramatically.

A client is diagnosed with mitral regurgitation. The nurse would expect to encounter which finding? A. Low red blood cell count B. Crushing chest pain C. Exertional dyspnea D. Elevated white blood cell count

C Fluid retention and diminished heart function cause exertional dyspnea in clients with mitral regurgitation as heart failure worsens. This is due to a rise in left atrial pressure and subsequent pulmonary and venous congestion.

The nurse is teaching a client about some of the side effects of fluoxetine. What information should the nurse be certain to include? A. Tachycardia, blurred vision, hypotension, anorexia B. Orthostatic hypotension, vertigo, hunger, reactions to tyramine-rich foods C. Drowsiness, dry mouth, changes in weight or appetite, reduced libido D. Photosensitivity, seizures, edema, hyperglycemia

C Fluoxetine (Prozac) is an antidepressant in a group of drugs called selective serotonin reuptake inhibitors (SSRIs). Commonly reported side effects include drowsiness and yawning, dry mouth, changes in weight or appetite, and sexual dysfunction; other reported side effects include insomnia and strange dreams, stuffy nose or tremors. People taking MAOIs should avoid foods containing tyramine. Tricyclic antidepressants (TCAs) can cause photosensitivity and TCA toxicity can cause hypotension and cardiac dysrhythmias.

A nurse is monitoring the contractions of a woman in labor. A contraction is recorded as beginning at 10:00 am and ending at 10:01 am. Another begins at 10:15 am. A third contraction begins at 10:30 am. What is the frequency of the contractions? A. 9 minutes B. 14 minutes C. 15 minutes D. 10 minutes

C Frequency is the time from the beginning of one contraction to the beginning of the next contraction.

A pregnant woman in the third trimester of pregnancy reports having severe heartburn. What is appropriate teaching by the nurse to help the woman alleviate these symptoms? A. Drink small amounts of liquids frequently B. Take sodium bicarbonate after each meal C. Sleep with head propped on several pillows D. Eat the evening meal just before retiring

C Heartburn is a burning sensation caused by regurgitation of gastric contents as a result of relaxed cardiac sphincter. It is best relieved by a slight upright sleeping position, eating small meals, and not eating within two to three hours of bedtime.

The nurse is monitoring oxygen saturation levels of preterm infants in the neonatal intensive care unit (NICU). Which of the following complications of oxygen therapy in preterm infants is the most common? A. Bronchopulmonary dysplasia (BPD) B. Necrotizing enterocolitis C. Retinopathy of prematurity (ROP) D. Hyperbilirubinemia

C In addition to birth weight and how early the infant is born, many factors contribute to the risk of ROP. But prolonged exposure to high concentrations of oxygen will cause irreversible damage to the eyes of preterm infants. Severe ROP is significantly reduced by keeping SpO2 levels stable and within narrow target ranges. BPD develops as a result of an infant's lungs becoming irritated or inflamed by mechanical ventilation, high levels of oxygen, infections, as well as heredity. Necrotizing enterocolitis and hyperbilirubinemia are problems associated with prematurity, but not oxygen therapy.

The nurse is developing a meal plan that would provide the maximum possible amount of iron for a child with anemia. Which dinner menu should be selected for the child? A. Peanut butter and jelly sandwich, apple slices, milk B. Fish sticks, french fries, banana, cookies, milk C. Ground beef patty, lima beans, wheat roll, raisins, milk D. Chicken nuggets, macaroni, peas, cantaloupe, milk

C Iron-rich foods include lean red meat, fish, egg yolks, green leafy vegetables, legumes, whole grains, nuts and dried fruits such as raisins. This dinner is the best choice because it is high in iron and is appropriate for toddlers who need finger foods.

During the initial injection of the contrast medium for an intravenous pyelogram (IVP) the client turns a ruddy red color and says, "I can't seem to catch my breath." What action should the nurse take next? A. Ask the client to take two slow deep breaths B. Ask the technician to slow the rate of administration of the contrast medium C. Administer epinephrine and stop administering the contrast medium D. Continue to assess vital skins and skin color

C Most acute severe adverse reactions to intravenous contrast media (ICM) occur within 20 minutes of the injection. Infusion of the ICM should be stopped. When the client is experiencing respiratory difficulty, epinephrine should be injected. The nurse should also prepare for oxygen administration at 10-12 L/minute and possible intubation.

The nurse is preparing a client for a kidney, ureter, bladder (KUB) radiograph test. Prior to the test, which action should the nurse plan to take for this client? A. Medicate the client with a PRN antihistamine prior to the examination B. Plan to have a fleets enema given prior to the examination C. Take no special actions before this examination D. Keep the client NPO for eight hours before the examination

C No special preparation is necessary for this examination; it is a simple x-ray.

The clinic nurse administers the influenza vaccine to a client. Within 15 minutes after the immunization is given, the client reports having itchy and watery eyes, increased anxiety, and difficulty breathing. What is the first action by the nurse in the sequence of care for this client? A. Direct the administration of diphenhydramine as ordered B. Monitor for hypotension with shock C. Administer epinephrine 1:1000 as ordered D. Initiate a patient airway

C Normally, the airway is a first priority. However, this client is talking to the nurse, so, the airway is patent. The next step is to realize the client is having a drug reaction. In the early stages of anaphylaxis, when the client has not lost consciousness and is normotensive, administering the 1:1000 concentration of epinephrine is first, and then applying the oxygen and watching for hypotension and shock are later actions. Diphenhydramine (Benadryl) is used to prevent allergic reactions.

After suffering a myocardial infarction (MI), the client is discharged with a prescription for a beta blocker drug. A nursing student asks why this drug is prescribed for someone who is not hypertensive. What is an appropriate response by the charge nurse? A. "A beta blocker will prevent orthostatic hypotension." B. "Most people develop hypertension after having had an MI." C. "This drug will decrease the workload on the client's heart." D. "Beta blockers increase the strength of heart contractions."

C One action of beta blockers is to decrease systemic vascular resistance by dilation of the arterioles. This is useful for clients with coronary artery disease and will reduce the risk of another MI or sudden death. Some of the more commonly prescribed beta blockers include: atenolol (Tenormin), bisoprolol (Zebeta), carvedilol (Coreg), and propranolol.

The nurse is providing information to clients taking chlorpromazine. Which teaching point should the nurse emphasize regarding this medication? A. Avoid canned citrus fruit drinks, especially grapefruit juice B. Avoid tyramine-containing foods C. Avoid direct sunlight D. Avoid foods fermented with yeast

C Phenothiazines increase sensitivity to the sun and makes clients especially susceptible to sunburn. This medication may also decrease clients' ability to sweat, making them more likely to get heat stroke. The nurse should recommend that clients treated with phenothiazines wear protective clothing when outside, to avoid tanning beds and to consistently use sunblock.

A 42 year-old male client diagnosed with hypertension tells the nurse he no longer wants to take propranolol. Which client statement best explains the reason why he does not want to take this medication? A. "I have difficulty falling asleep." B. "I'm having problems with my stomach." C. "I'm experiencing decreased sex drive." D. "I feel so tired all the time."

C Propranolol is a beta-blocker used to treat many conditions, such as essential tremors, angina, hypertension, and heart rhythm disorders. Common side effects of this drug include nausea, diarrhea, constipation, stomach cramps, rash, tiredness, dizziness, sleep problems, and vision changes. Additionally, propranolol may cause decreased sex drive, impotence or difficulty having an orgasm in men. The clients can be switched to an alternative antihypertensive, such as an angiotensin-converting enzyme (ACE) inhibitor or a calcium channel blocker.

A client has been receiving lithium for the past two weeks for the treatment of bipolar illness. When planning client teaching, what is most important for the nurse to discuss with the client? A. Take a diuretic with lithium and avoid excessive fluid intake B. Focus more on controlling the feelings of depression initially, and not the mania C. Come in for regular evaluation of serum lithium levels D. Maintain a heart-healthy, low-sodium diet

C The client should understand that it is most important to evaluate the serum lithium levels. The client should drink plenty of fluids; not drinking enough will cause lithium levels to rise. Client should understand that they may need more fluids when it's hot outside or during exercise. Because lithium is a salt, the client should not be on a low-salt diet without first discussing this with the health care provider. Less salt can cause lithium levels to rise and result in shakiness, muscle weakness, slurred speech, and/or nausea and vomiting. Clients should be cautioned to report any symptoms of mood instability.

A client in labor wishes to have an epidural. The nurse reviews the client's history and laboratory results. Which of the following findings is a contraindication to the client receiving an epidural? A. Sodium is 138 mg/dL (138 mmol/L) B. Hemoglobin is 11.2 g/dL (6.95 mmol/L) C. Platelet count is 95,000/mcL D. White blood cell count is 8,000/mcL

C The hemoglobin is low, but anemia is common in pregnancy. The sodium level and white blood cell count are within normal limits. Normally there is a decrease in platelets during pregnancy but this client's platelet count is significantly reduced and places the client at risk for bleeding; therefore, the epidural is contraindicated for this client. Thrombocytopenia in pregnancy can be defined as a platelet count less than 116,000 per microliter (in nonpregnant individuals, thrombocytopenia is usually defined as a platelet count less than 150,000 per microliter.)

A client with asthma has low-pitched wheezes present on the final half of exhalation. One hour later the wheezing is higher pitched and extends throughout exhalation. What should this change in assessment finding indicate to the nurse? A. The airway obstruction has improved B. The client is hyperventilating C. The airway obstruction has increased D. The client needs to be suctioned

C The higher pitched a sound is, the more narrow the airway. Therefore, the airway obstruction has increased or worsened, necessitating urgent intervention to promote bronchodilation or a change in interventions that are not be effective. Other signs of worsening asthma and the development of respiratory distress would include restlessness, anxiety, increased pulse and blood pressure and increased respiratory rate with the use of accessory muscles of breathing. Although secretions may be thick and tenacious during an acute asthmatic attack, there is no data to support the need for suctioning in this question.

A nurse is assessing a client with a deep vein thrombosis. Which finding should the nurse anticipate? A. Chest pain B. Bilateral ankle edema C. Swelling of one lower extremity D. Rapid respirations

C The most common signs of deep vein thrombosis are pain in the region of the thrombus and unilateral swelling of the extremity distal to the site. Bilateral pedal edema is usually associated with fluid overload in right heart failure or venous insufficiency. Chest pain and tachypnea that occurs suddenly in the client with DVT are signs of possible pulmonary embolism, a serious complication of DVT.

The nurse is reviewing medications with a client diagnosed with heart failure. The client asks the nurse how much longer he has to take the prescribed diuretic. Which is the best response by the nurse? A. "Please talk to your health care provider about medications and treatments." B. "You will have to take this medication for about a year." C. "The medication must be continued as long as the fluid problem needs to be controlled." D. "As you urinate more, you will need less medication to control fluid."

C The most therapeutic response is the one that addresses the client's health condition and gives the client accurate information, which is that he will take the medication for as long as needed to treat his medical condition. The nurse should determine if the client understands why he is taking this and any other medications and to reinforce teaching on possible side effects or adverse effects and when to notify the health care provider.

The nurse is organizing play for a small group of hospitalized children. Which playroom activity is appropriate for a group of 7 year-old children? A. Chess and television programs B. Finger paints and water play C. Board games with rules D. "Dress-up" clothes and props

C The purpose of play for the 7 year-old is the development of cooperation through team play or joining groups such as the Boy Scouts or Girl Scouts. Rules are a focus in this age group. Logical reasoning and social skills are developed through play.

The nurse is assigned to care for a client diagnosed with HIV/AIDS. A first-semester nursing student asks the nurse how a diagnosis of AIDS is determined, other than a positive HIV test. What response by the nurse is the best explanation for how AIDS is diagnosed? A. "The presence of any number of opportunistic infections and testing positive on the viral load test." B. "Having symptoms of anxiety, dementia, depression and insomnia, along with a low viral load." C. "Having a CD4+ lymphocyte count less than 400 and a positive Western Blot test." D. "Having a CD4+ lymphocyte count less than 200 and one or more opportunistic infections."

D A CD4+ lymphocyte count is normally 600 to 1000 cells per cubic millimeter of blood. The Centers for Disease Control defines AIDS as someone who has a positive HIV blood test, one or more opportunistic infections (such as candidiasis and Kaposi's sarcoma) and a CD4+ lymphocyte count of less than 200. The ELISA Test is used to detect HIV infection; the Western Blot test is used to confirm a positive ELISA test. A viral load test measures the amount of virus in the blood; individuals with higher viral loads are at greatest risk for progressing from HIV infection to AIDS.

Two hours after the normal spontaneous vaginal delivery of a woman who is gravida 4 para 4, the nurse notes that the fundus is boggy and displaced slightly above and to the left of the umbilicus. What should be the initial nursing action? A. Assess lochia for color and amount B. Monitor pulse and blood pressure C. Call the health care provider immediately D. Ask the client to empty the bladder

D A full bladder can displace the uterus and prevent contraction. After the woman empties the bladder, the fundus should be assessed again.

The nurse is assigned to care for four clients. Based on the diagnoses and histories, which client should be assessed first after listening to report? A. The client with asthma who is now ready for discharge B. The client with chronic renal failure who just returned from dialysis C. The client with pancreatitis who was admitted yesterday D. The client with peptic ulcer disease who has been vomiting all day

D A perforated peptic ulcer may result in nausea, vomiting and abdominal distention or board-like abdomen. This might be a life-threatening situation. The client should be assessed immediately for initial findings of shock with notification of the health care provider. The other clients would be considered stable with minimal risk of an emergency.

A client has been admitted with a diagnosis of bacterial meningitis. In reviewing the laboratory analysis of cerebrospinal fluid (CSF), the nurse should expect to see which result? A. Increased glucose levels B. Elevated sedimentation rate C. Clear cerebrospinal fluid D. High protein levels

D A positive CSF for bacterial meningitis would include the presence of protein, a positive blood culture, decreased glucose, cloudy color with an increased opening pressure, and an elevated white blood cell count. If it was viral meningitis, the difference would be that the CSF glucose would be within normal parameters.

An 8 year-old is admitted to the hospital for surgery. The child's parent reports the allergies listed below. Which of these allergies should alert the staff to implement system safeguards? A. Shellfish B. Mold C. Perfumed soap D. Balloons

D A reaction to balloons or rubber toys may indicate a latex allergy; children with a history of allergies to avocados, bananas, and kiwi may also be at risk for latex allergy. Common hospital items that may contain latex include catheters and other tubing, stethoscopes, blood pressure cuffs, and even wheelchair tires. All personnel who have contact with the child must be aware of this allergy and use latex-free supplies and equipment. If the child is in a semiprivate room, latex precautions should be used for both clients.

A nurse is working in the emergency department. As a general guide for emergency management of acute alcohol intoxication, which information from the lab would be the priority before treatment can begin? A. Albumin B. Hemoglobin C. Glucose D. Ethanol/alcohol

D A verbal report of alcohol consumption by the client is notoriously inaccurate (usually on the lower side of ingestion.) So, blood alcohol levels are initially obtained to determine the level of intoxication. The amount of alcohol consumed determines how much medication the client needs for detoxification and treatment. The other lab values are important but are not the priority.

The client who is receiving intermittent enteral nutrition through a gastrostomy tube has had four diarrhea stools in the past 24 hours. What approach should the nurse now take? A. Increase the amount of water used to flush the tube B. Increase the infusion rate when the formula is running C. Attach a rectal bag to protect the skin D. Review the medications the client is receiving

D Antibiotics and medications containing sorbitol may induce diarrhea. An increased rate often results in increased peristalsis and diarrhea. Attaching a rectal bag may be needed, but this does not address the problem.

A nurse is speaking to a group of retired people about glaucoma. Which comment by one of the retirees is correct and should be reinforced by the nurse? A. "I wish I didn't have to stop driving. I don't know how I'll manage." B. "I take half of the usual dose for my sinuses to maintain my blood pressure." C. "I have to sit at the side of the pool with the grandchildren since I can't swim with this eye problem." D. "I take extra fiber and drink lots of water to avoid getting constipated."

D Any activity that involves straining increases intraocular pressure and should be avoided in clients with glaucoma. Since antihistamines can increase pressure in the eye, they should be avoided. Many people with glaucoma do give up driving at night, but most people do not have to give up driving altogether. Swimming should cause no problems (as long as the person has not had recent eye surgery) but it might be a good idea to wear goggles when in a pool.

The nurse is admitting a client diagnosed with chronic bilateral glaucoma and asks about the current health care issues. Which of the following statements made by the client would the nurse anticipate? A. "I have constant blurred vision." B. "I can't see on my left side." C. "I have specks floating in my eyes." D. "I have to turn my head to see around my room."

D As intraocular pressure becomes elevated in chronic glaucoma, there is a slow, progressive loss of the peripheral visual field in the affected eye(s); if untreated or uncontrolled, it eventually can lead to blindness. Tiny, painless particles floating inside the eye that are called floaters; retinal detachment can also include floaters but also sparks or flashes of light. Blurred vision can have many causes, including refractive errors, chronic dry eyes, cataracts and macular degeneration.

The client is newly diagnosed with type 1 diabetes mellitus. Which of these approaches would be the best strategy for the nurse to use when teaching insulin injection techniques? A. Give written pre and post tests B. Allow another diabetic to assist C. Ask questions during practice D. Observe a return demonstration

D Because this is a psychomotor skill, observation of the client doing the task is the best way to know if the client has learned the proper technique.

The nursing team consists of a registered nurse (RN), licensed practical nurse (LPN) and a certified nursing assistant (CNA). When making assignments, which team member could be assigned to measure and document vital signs for medically stable clients? A. Licensed Practical Nurse B. The charge nurse C. Registered Nurse D. Certified nursing assistant

D Certified nursing assistants perform routine tasks that have predictable outcomes, which is why CNAs can be assigned to measure and document vital signs for medically stable clients. Before making this assignment, the charge nurse must know that the CNA has received the appropriate training and is competent to perform the activity. Also, a fully-qualified nurse must be available to provide supervision during the performance of any assigned task.

The nurse is planning care for a client with a cerebral vascular accident (CVA). Which approach would be most effective in the prevention of skin breakdown? A. Place client in the wheelchair for four hours daily B. Pad the bony prominences C. Massage reddened bony prominence D. Reposition every two hours when in bed

D Clients who are at risk for skin breakdown develop fewer pressure ulcers when turned every two hours. By relieving the pressure over bony prominences at frequent scheduled intervals, blood flow to areas of potential injury is maintained. If the client is in a wheelchair, a shift of the weight should be done every hour. Massage of reddened bony prominences is no longer recommended as a result of evidence-based research.

The nurse in a long-term care facility is assigned to the dementia unit. What type of functions should the nurse expect to see impaired in these clients? A. Balance, flexibility and coordination B. Hearing, speech and sight C. Endurance, strength and mobility D. Learning, creativity and judgment

D Dementia is not a single disease but a general term used to describe symptoms such as impairments to memory, communication and thinking. There are many causes of dementia and although we generally associate dementia with aging, we know that it is due to degenerative changes to the brain. The other options include other expected changes due to aging, but do not necessarily indicate cognitive impairment related to dementia.

The nurse is consulting with a nutritionist regarding an appropriate diet for a client recently diagnosed with renal disease. Select the most appropriate diet for the client with renal disease. A. High potassium, high fat, low protein B. High protein, high fat, high carbohydrate C. High carbohydrate, high protein, moderate fat D. Restricted protein, low sodium, low phosphorus

D Dietary modification is important with renal disease. Protein intake should be limited to decrease nitrogenous waste production (typically, 1 gram of protein per kilogram of body weight per day is recommended). The client should also follow a sodium, potassium and phosphorous-restricted diet.

A nurse has been teaching an apprehensive primipara who has had initial difficulty in nursing the newborn. What observation at the time of discharge suggests that initial breast-feeding is effective? A. The newborn refuses the supplemental bottle of glucose water B. The newborn falls asleep after three minutes at the breast C. The mother awakens the newborn to feed whenever it falls asleep D. The mother feels calmer and talks to the baby while nursing

D Early evaluation of successful breast-feeding can be measured by the client's voiced confidence and satisfaction with the neonate.

A client has been on antibiotics for 72 hours to treat cystitis. Which findings reported by the client require priority attention by the nurse? A. Smelly urine B. Nausea and anorexia C. Burning on urination D. Elevated temperature

D Elevated temperature after 72 hours on an antibiotic indicates that the antibiotic has not been effective in eradicating the offending organism. The health care provider should be informed immediately so that an appropriate medication can be prescribed and complications, such as pyelonephritis, are prevented. The smelly urine and burning are expected with cystitis and during initial treatment. Gastrointestinal findings may be related to the antibiotics as a side effect and should also be reported. However, they are a lower priority and may resolve if the antibiotic is changed.

A client diagnosed with rheumatoid arthritis is prescribed etanercept. What will the nurse include in the discharge teaching about etanercept? A. "Take the medication daily, first thing in the morning on an empty stomach." B. "The medication needs to be mixed well. You can shake the bottle to mix it." C. "You will need to come into the clinic every 6 weeks to receive an intravenous infusion." D. "If you keep the medication in a refrigerator, be sure to allow it to warm to room temperature before injecting it."

D Etanercept is in a class of medications called tumor-necrosis factor inhibitors and is used alone or with other medications to relieve the symptoms of some autoimmune disorders. It usually comes in a prefilled syringe and an automatic injection device. The medication is injected subcutaneously once a week. Besides knowing how and where to inject the medication, the client should be instructed never to shake the vial and, if the medication has been refrigerated, the nurse should reinforce that the client should simply place the medication on a flat surface (like a countertop) and allow it to warm to room temperature for about 30 minutes but never heat it in a microwave or place it in hot water.

Parents of a 7 year-old child call a community clinic nurse because their daughter was sent home from school because of a rash. The child had been seen the day before by the health care provider and was diagnosed with fifth disease (erythema infectiosum). What is the most appropriate action by the nurse? A. Inform the teacher that the child is receiving antibiotics for the rash B. Tell the parents to bring the child to the clinic for further evaluation C. Refer the school officials to printed materials about this viral illness D. Explain that this rash is not contagious and does not require isolation

D Fifth disease is a viral illness with an uncertain period of communicability (perhaps one week prior to and one week after onset). Isolation of the child with fifth disease is not necessary except in cases of hospitalized children who are immunosuppressed or have aplastic crises. The parents may need written confirmation of this from the health care provider to give to the teacher.

Which of these statements by the nurse reflects the best use of therapeutic interaction techniques? A. "I'd like to know more about your family. Tell me about them." B. "I understand that you lost your partner. I don't think I could go on if that happened to me." C. "You look very sad. How long have you been this way?" D. "You look upset. Tell me what you are feeling right now."

D Giving broad opening statements and making observations are examples of therapeutic communication. To ask about the client's family is not supported by any assessment data provided. Therefore, this approach would not be therapeutic in the absence of a reason to inquire about the client's family. To comment about a lost partner is incorrect because of an inappropriate personal remark by the nurse. To make a comment of the client looking sad is appropriate. However, to ask about the narrow focus of "how long have you been this way" is not as therapeutic as the correct option. The correct answer offers the client a broad opportunity to talk about concerns.

A client telephones the community clinic to ask about a home pregnancy test she used this morning. The nurse should understand that the presence of which hormone strongly suggests that a woman is pregnant? A. Estrogen B. Alpha-fetoprotein C. Progesterone D. Human chorionic gonadotropin

D Human chorionic gonadotropin (HCG) is the biologic marker on which pregnancy tests are based. Reliability is about 98%, but the test does not conclusively confirm pregnancy. Progesterone is the hormone that maintains a pregnancy and when its levels drop the woman will go into labor.

A client has an order for antibiotic therapy after hospital treatment of a staph infection. Which point should the nurse emphasize? A. Schedule follow-up blood cultures B. Monitor for signs of recurrent infection C. Visit the provider in a few weeks D. Complete the full course of medications

D In order for antibiotic therapy to be effective in eradicating an infection, the client must compete the entire course of prescribed therapy. If the client were to stop taking the medication when the findings subside, it could lead to a recurrence of the infection and/or drug resistance to any subsequent antibiotic therapy.

A 24-year old female calls the health clinic and informs the nurse that she has missed two periods while using a 21-day hormone-containing contraceptive. She states that she may have forgotten to take a pill for a few days in one cycle. What is the most appropriate response by the nurse? A. Come to the clinic and discuss different options for contraception that do not require taking a daily pill B. Immediately stop taking the birth control pills and make an appointment to rule out a pregnancy C. Take the over-the-counter emergency contraceptive levonorgestrel (Plan B) D. Continue taking the pills, take a home pregnancy test and call the health care provider for advice about the test results

D It's not unusual for women who are taking hormone pills for birth control to have light periods or no bleeding at all. If a woman skips two periods in a row, regardless if she missed any pills, she should take a home pregnancy test and call the health care provider (HCP) for advice about the test results. She should not stop taking the pills and risk a pregnancy unless instructed to do so by the HCP. Plan B is a type of emergency contraception, also known as the "morning after pill" and should be used within 72 hours after unprotected sex; it will not stop development of a fetus once someone is already pregnant. Switching to a different form of contraception is a possibility, but only after the woman confirms she is not pregnant.

The nurse observes a family member administer a rectal suppository. The family member turns the client to lie on the left side, pushes the suppository in with one finger up to the second knuckle, removes the finger, and then waits 10 minutes before turning the client to the right side. What is the appropriate comment for the nurse to make? A. "Why don't we now have the client turn back to the left side." B. "Let's check to see if the suppository is in far enough." C. "Did you feel any stool in the intestinal tract?" D. "That was done correctly. Did you have any problems with the insertion?"

D Left side-lying position is the optimal position for clients to receive rectal medications. Due to the position of the descending colon, left side-lying allows the medication to be inserted and move along the natural curve of the intestine and facilitates retention of the medication. The suppository should be somewhat melted after 10 to 15 minutes and the client can move into any position of comfort. There is no data in the stem to support any of the other options.

The client was admitted two days ago with a diagnosis of myocardial infarction (MI). When assessing this client, the nurse notes the client's temperature is now 101.1 F (38.5 C). What is the most appropriate nursing intervention? A. Call the health care provider immediately B. Increase the client's fluid intake C. Send blood, urine and sputum for cultures D. Administer PRN acetaminophen as ordered

D Leukocytosis and fever are common outcomes on day two after a heart attack because of the inflammatory process associated with an acute MI. Nursing interventions should focus on the promotion of comfort.

A nurse is caring for a client who is receiving methyldopa. Which assessment finding would indicate to the nurse that the client may be having an adverse reaction to the medication? A. Hyperkalemia B. Palpitations C. Nausea D. Sedation

D Methyldopa (Aldomet) is used to treat hypertension. The nurse should assess the client for alterations in mental status, such as sedation. Other common side effects are dizziness, dry mouth, headache and weakness. These changes should be reported to the health care provider.

The nurse is assessing a client who states her last menstrual period was March 16, and she has missed one period. The client reports episodes of nausea and vomiting. Pregnancy is confirmed by a urine test. What should the nurse calculate as the estimated date of delivery (EDD)? A. April 8 B. February 11 C. January 15 D. December 23

D Naegele's rule states: Add seven days and subtract three months from the first day of the last regular menstrual period to calculate the estimated date of delivery.

During a discussion with the nurse manager, a staff nurse confides that she is attracted to a client regularly assigned to her. Which of the following actions should be implemented following this discussion? A. The nurse reassigns all personal care of the client to the nursing assistant B. The nurse waits until after discharge to tell the client about her feelings C. The nurse continues to provide care for the client D. The nurse transfers the care of the client to another nurse

D Nurses must practice in a manner consistent with professional standards and be knowledgeable about professional boundaries. A nurse's challenge is to be aware of feelings and to always act in the best interest of the client, avoiding inappropriate involvement. In this case, the nurse did all the right things - aware of her feelings, she consulted with her supervisor and together they decided it would be best if this client were no longer assigned to this nurse. If the nurse had acted on her feelings, this would have been a boundary violation and she could have been subject to board of nursing disciplinary action.

A nurse is caring for a client postop after a construction of a permanent colostomy. The client begins to cry, saying "I'll never be attractive again with this ugly red thing." What should be the first action taken by the nurse? A. Invite the partner to participate in colostomy care after viewing an instructional video B. Suggest sexual positions that hide the colostomy C. Arrange a consultation with a sex therapist experienced in working with colostomy clients D. ncourage the client to discuss feelings about the colostomy

D One of the greatest fears of clients with a colostomy is the fear that sexual intimacy is no longer possible. However, the client's personal feelings about the stoma and colostomy care, as well as the client's specific concerns, need to be assessed to accurately identify the problem(s) to be solved. An assessment should occur before specific suggestions for dealing with the sexual concerns are given.

A couple attempting to conceive asks the nurse when ovulation occurs. The woman reports a regular 32-day cycle. Which response by the nurse is correct? A. Days 11-13 B. Days 7-10 C. Days 14-16 D. Days 17-19

D Ovulation occurs 14 days prior to menses. Considering that the woman's cycle is 32 days, subtracting 14 from 32 suggests ovulation is at about the 18th day.

The nurse is caring for a client with acute pancreatitis. After pain management, which intervention should be included in the plan of care? A. Place the client in contact isolation B. Institute seizure precautions C. Provide a diet high in protein D. Encourage the client to cough and deep breathe every two hours

D Respiratory infections are a common complication of pancreatitis because fluid in the retro-peritoneum can push up against the diaphragm, causing shallow respirations. Coughing and deep breathing every two hours will diminish the occurrence of this complication. The other interventions are not appropriate, and the client will be NPO during the initial period of treatment for pancreatitis.

A nurse checks a client who was placed on a volume-cycled ventilator. Which finding indicates that the nurse needs to suction the client? A. Report of nausea B. Heart rate 82 BPM C. Drowsiness D. Restlessness

D Restlessness, increased heart and respiratory rates and noisy auditory expiration suggest hypoxia. These are indications for suctioning.

The RN has just admitted a client newly diagnosed with severe depression. What domain should be a priority focus as the nurse identifies the applicable nursing diagnoses? A. Activity B. Elimination C. Nutrition D. Safety

D Safety is a care priority for all inpatient clients, and a depressed client is at acute risk for self-destructive behavior. Precautions to prevent suicide must be a part of the nursing care plan. Note that the client has severe depression, which requires higher safety needs.

The nurse is assessing an infant with developmental dysplasia of the hip. Which finding should a nurse anticipate? A. Diminished femoral pulses B. Symmetrical gluteal folds C. Unlimited hip abduction D. Unequal leg length

D Shortening of the affected leg is a sign of developmental dysplasia of the hip. Other signs of hip dysplasia in an older infant include limited hip abduction and asymmetric gluteal skin folds. An ultrasound examination is typically used to confirm developmental dysplasia of the hip in the young infant; x-rays are used when the infant is older than 3 months.

A client has a nasogastric tube after colon surgery. Which of these tasks can be safely assigned to an unlicensed assistive person (UAP)? A. Irrigate the nasogastric tube with the ordered solution B. Monitor the client for nausea or other gastric complications C. Monitor the type and amount of nasogastric tube drainage D. Perform nostril and mouth care every two hours or as ordered

D Skin care around a nasogastric tube and oral hygiene are routine tasks with expected outcomes - this is an appropriate assignment for the UAP. The other tasks or activities would assigned to nurses because they require specialized nursing knowledge, skill or judgment.

The client is confirmed to have a diagnosis of cerebral vascular accident. What serum lab value should the nurse check prior to administration of alteplase (TPA)? A. Arterial blood gases and complete blood count B. Blood urea nitrogen and creatinine C. Potassium and magnesium D. Prothrombin time and activated partial thromboplastin time

D TPA is a potent thrombolytic enzyme. Because bleeding is the most common side effect, the nurse must evaluate clotting studies prior to administration.

A child and the family were exposed to Mycobacterium tuberculosis about two months ago. For confirmation of the presence or absence of an infection in their system, it is important for all family members to have which test? A. Blood culture and sensitivity B. Sputum culture and sensitivity C. Chest x-ray without contrast D. Purified protein derivative (PPD) test

D The administration of the purified protein derivative (PPD) intradermal test determines if someone has developed an immune response to the bacterium that causes tuberculosis (TB). The response occurs if someone currently has TB or was exposed to it. An incubation period of 2 to 12 weeks after exposure is required before the PPD test can be positive. A positive skin test is when someone has a 10 mm or larger induration (for immunocompromised clients, a positive test is when the induration is 5 mm or larger.) If the client has a positive skin test, a CXR is ordered and if this shows signs of TB, a sputum sample is ordered.

The nurse is evaluating the growth of a 12 month-old child. Which finding would the nurse expect to be present in the child? A. Head is greater than the chest circumference B. Increased 10% in height C. Two deciduous teeth D. Tripled the birth weight

D The birth weight usually triples by the end of the first year of life. Height usually increases by 50% from birth length. A 12 month-old child should have approximately six teeth; estimate the number of teeth by subtracting 6 from the age in months (12 - 6 = 6). By 12 months of age, head and chest circumferences are approximately equal.

A client who is scheduled for antineoplastic chemotherapy tells the nurse, "I have a fear of being sick all the time. I would like to try acupuncture." Which of these statements made by the client is incorrect about acupuncture and needs clarification by the nurse? A. "Some needles go as deep as three inches, depending on where they're placed in the body and what the treatment is for. The needles usually are left in for 15 to 30 minutes." B. "In traditional Chinese medicine, imbalances in the basic energetic flow of life — known as qi or chi — are thought to cause illness." C. "By inserting extremely fine needles into the acupuncture points, it is believed that energy flow will rebalance and the body's natural healing mechanisms will take over." D. "The flow of life is believed to flow through major pathways called nerve clusters in your body."

D The major pathways are called meridians, not nerve clusters. The other options offer correct information about acupuncture.

The nurse is making rounds at the beginning of the shift and asks how each client is feeling. Which statement made by a client would require immediate action by the nurse? A. "The pain came on after dinner. That soup seemed very spicy." B. "When I take in a deep breath, it stabs like a knife." C. "When I turn in bed to reach the remote for the TV, my chest hurts." D. "I feel pressure in the middle of my chest like an elephant is sitting on my chest."

D This is a classic description of chest pain in men caused by myocardial ischemia, requiring immediate assessment and intervention to prevent possible damage to the heart muscle. Pain after spicy food is often the result of irritation and gastric indigestion. The pain with a deep breath is typically from an inflammation of the pleural covering of the lung, called pleurisy. Pain with movement of the chest, such as turning in bed, is typically caused by costochondritis, which is inflammation of the cartilage between the ribs and the sternum, and can be reproduced by palpation of the the painful area.

The nurse is caring for clients diagnosed with personality disorders. Which of these therapies best describes an intervention to deal with the behaviors of such clients? A. Point out inconsistencies in speech patterns to correct thought disorders B. Accept the client and the client's behavior unconditionally C. Encourage dependency in order to develop ego controls D. Set consistent limits to be enforced around-the-clock

D Treatment approaches for personality disorder diagnosis include to restructure the personality, assist the person with the advancement of developmental levels and set limits for maladaptive behavior such as acting out. Very few behaviors are to be accepted unconditionally in any circumstance. Dependency is unlikely an outcome for most situations.

The oncology client is using patient controlled analgesia (PCA) with morphine for pain control. The client reports having pain and states it is a 7 (on a scale of 0 to 10). Indicate the correct sequence of nursing interventions by dragging and dropping the sentences in the correct order. A. Consult with the health care provider B. Confirm that there is power to the pump and the tubing is patent C. Check the chart for orders for treating breakthrough pain D. Ensure the client is using the PCA equipment properly E. Assess the level of consciousness and respiratory status

D,B,E,C,A The nurse must ensure that the client understands how to use the PCA and should have the client demonstrate how to push the button. If indeed the client is using the PCA properly, then the nurse must ensure the machine is mechanically sound, i.e., the power is on, tubing connected and not kinked, etc. Because narcotics can cause sedation and respiratory suppression, the nurse must determine that these are not problems. The nurse can then check to see if there is an order for breakthrough pain. The last step would be to consult with the health care provider, possibly for further orders.


Ensembles d'études connexes

Chapter 26: The Reproductive System

View Set

Chapter 47: Caring for Clients with Disorders of the Liver, Gallbladder, or Pancreas

View Set

Chapter 62: Caring for Clients with Traumatic Musculoskeletal Injuries

View Set

Microeconomics Chapter 24: Monopoly

View Set

Abnormal Psychology Pract. Ch. 16

View Set

Chapter 5: Cultural Diversity PrepU

View Set